LSAT and Law School Admissions Forum

Get expert LSAT preparation and law school admissions advice from PowerScore Test Preparation.

General questions relating to LSAT Logical Reasoning.
 elbism
  • Posts: 26
  • Joined: Mar 21, 2016
|
#22583
Hi!

I would just like a little area of confusion cleared up for me, if it's no trouble.

In 2nd family questions, such as assumption qs, where the stem tells us to believe the answer choice as true, and the stimulus says

"A is B"

and an answer choice says

"A is not B"

Then what are we to believe? The reason I ask this is because i've seen several questions where the answer choice "A is not B" was the INCORRECT answer, and people have explained that this is because the stimulus clearly states A is B, so 'A is not B' is irrelevant. But if we are taking the answer choices to be true then doesn't that trump the leverage of the premises? I just want to know how can we judiciously discern between the implied veracity of the stimulus v. the answer choices when we are told to believe the answer choices are true (obviously that can only be to a certain extent otherwise the entire stimulus would become redundant?)
 Nikki Siclunov
PowerScore Staff
  • PowerScore Staff
  • Posts: 1362
  • Joined: Aug 02, 2011
|
#22584
Hi elbism,

In the Help and Hurt families of questions (comprised predominantly of Strengthen, Justify, Resolve, and Weaken questions), you are indeed asked to assume that each answer choice is a true statement. This is clearly stated in the question stem, which usually takes the form of "Which one of the following, if true, ..." So, the mere fact that an answer choice might contradict a statement made in the stimulus is not, by itself, sufficient to disqualify that answer choice. Of course, such a contradiction may ultimately prove irrelevant, and thus the answer choice could be incorrect, but the mere fact that it exists has no bearing on the accuracy of that answer. This is in direct contrast to questions in the Prove and Disprove families, where a factual inconsistency with the information contained in the stimulus would be dispositive, and sufficient to eliminate that answer choice from consideration.

The only exception to this line of reasoning concerns how we handle Assumption questions. While technically in the Help family, the Assumption stem does not dictate that you accept each answer choice as true. Instead, it directs you to identify a statement upon which the argument in the stimulus depends. In other words, if the conclusion in the stimulus is to be valid, which one of the following must also be assumed to be true? In many ways, Assumption questions are closer to the Prove than to the Help family of questions. Consequently, any inconsistency with the facts presented in the stimulus would immediately render an Assumption answer choice incorrect, just as it would with any Must Be True question. For more information about the similarities and differences between MBT and Assumption questions, check this blog post out.

Hope this helps a bit!
 elbism
  • Posts: 26
  • Joined: Mar 21, 2016
|
#22585
Nikki Siclunov wrote:Hi elbism,

In the Help and Hurt families of questions (comprised predominantly of Strengthen, Justify, Resolve, and Weaken questions), you are indeed asked to assume that each answer choice is a true statement. This is clearly stated in the question stem, which usually takes the form of "Which one of the following, if true, ..." So, the mere fact that an answer choice might contradict a statement made in the stimulus is not, by itself, sufficient to disqualify that answer choice. Of course, such a contradiction may ultimately prove irrelevant, and thus the answer choice could be incorrect, but the mere fact that it exists has no bearing on the accuracy of that answer. This is in direct contrast to questions in the Prove and Disprove families, where a factual inconsistency with the information contained in the stimulus would be dispositive, and sufficient to eliminate that answer choice from consideration.

The only exception to this line of reasoning concerns how we handle Assumption questions. While technically in the Help family, the Assumption stem does not dictate that you accept each answer choice as true. Instead, it directs you to identify a statement upon which the argument in the stimulus depends. In other words, if the conclusion in the stimulus is to be valid, which one of the following must also be assumed to be true? In many ways, Assumption questions are closer to the Prove than to the Help family of questions. Consequently, any inconsistency with the facts presented in the stimulus would immediately render an Assumption answer choice incorrect, just as it would with any Must Be True question. For more information about the similarities and differences between MBT and Assumption questions, check this blog post out.

Hope this helps a bit!
So you're saying that in, for example, a weaken question.. an answer choice that directly contradicts the stimulus could be the correct answer?

Let me just give an example to elucidate my confusion a little better.
Say for example the stimulus says

"Jack should go to school today, because then he will have a good day"
and the question asks us to weaken the statement,
and an answer choice suggests

"Jack should not go to school today if he's going to have a bad day"

I mean, that would clearly contradict the stimulus and it seems way too obvious to be correct, but at the same time, i can't logically disqualify it as the correct answer... and that's where i think i'm stuck. i suppose an answer that said

"jack should not go to school today, if he's going to get bullied"
Would make more sense as logically weakening the stimulus, but didn't the stimulus say he was going to have a good day? so doesn't it make the possibilities of all other types of days that he could have redundant?

I do understand that we are supposed to take the answer choices as true, as stated "if true..." but usually those answer choices are assumptions that could be true but are not even mentioned in the stimulus so don't have any particular baring on directly contradicting the main conclusion. I've seen this happen in a couple of questions and i'm always confused as to how to go about elimination. Every time i've seen it, the contradicting answer has been incorrect, and that's essentially why i'm asking, because it would seem to allude to the idea that we should take the answer as true only to the extent that it doesn't contradict the stimulus (if the contradicting answer was correct it would make sense as a correct answer and i would not be confused)
 Robert Carroll
PowerScore Staff
  • PowerScore Staff
  • Posts: 1819
  • Joined: Dec 06, 2013
|
#22588
elbism,

With a Weaken question, contradicting the stimulus is perfectly fine; saying that the author simply got something wrong, or misstated the facts, or failed to consider a limitation on the evidence given, is a great way to weaken the argument.

If a stimulus said "Jack should go to school, because if he does, he will have a good day," it could be weakened by an answer that said "Jack will have a bad day if he does to school today." There is nothing wrong with contradicting something the stimulus stated. Because the falsity of that statement would make the conclusion ("Jack should go to school") more suspect than it was before, the new information in the answer choice is doing exactly what it ought to do in order to weaken the argument in the stimulus.

That is a situation with a direct contradiction of evidence in the stimulus. You may instead have a situation where evidence in the stimulus is made less likely to be true by new information in the answer choice. Thus, "Jack will be bullied if he goes to school today." This does not contradict the statement "Jack will have a good day if he goes to school today," because a day may end up good overall even if it has some bad elements. However, all other things being equal, a day involving bullying is worse than one without bullying, so the fact that he will be bullied gives us some reason to think his day will not be good.

Remember that the stimulus in a Third Family question contains information that can be suspect. The question is not instructing you to trust all that information (it tells you to trust the answer choices), so even if the stimulus said he was going to have a good day, you aren't committed to having to believe the author of the stimulus that this is true. Thus, conflicting or flatly contradictory information is permissible in the answer choice.

Contradictory information is permissible in a Weaken question because of the fact that new information is permissible - the question asks which of the answers, if true, would weaken. Contradictory information is desirable for a Weaken question because it asks which answer weakens the argument, and contradicting information in the stimulus often contradicts premise facts that were supporting the argument. Showing them to be false, or simply to be less likely to be true, undermines the support for an argument.

Note the two words I used - permissible and desirable. For something in the Second Family, like a Strengthen question, new information is permissible, so information that contradicts the stimulus is permissible. However, it's difficult to see how contradictory information would be desirable for something in the Second Family. Thus, while being contradictory is not per se disqualifying for an answer choice, the answer choice has to contain something that supports the argument as well, and this is just something you won't find in the same answer choice.

If you can give specific examples, that would be great!

Robert Carroll
 Nikki Siclunov
PowerScore Staff
  • PowerScore Staff
  • Posts: 1362
  • Joined: Aug 02, 2011
|
#22589
Hi elbism,

Let me add to Robert's excellent response above.

I'm afraid that neither the example you gave, nor your reasoning behind it, are logically consistent. If all the argument stated is this:
Jack should go to school today, because then he will have a good day
... then the only logically valid ways to weaken that argument would be to show one of the following:

1. Counterexamples where Jack has gone to school, but he had a bad day;
2. Counterexamples where Jack didn't go to school, but he had a blast: perhaps Jack has even better days when he stays home and plays video games all day long?

Neither of the answer choices you propose have any bearing on the argument at hand. Neither contradicts the stimulus, and both are irrelevant to the conclusion.

Last, I need to comment on something you said in your last paragraph:
I do understand that we are supposed to take the answer choices as true, as stated "if true..." but usually those answer choices are assumptions that could be true but are not even mentioned in the stimulus so don't have any particular baring on directly contradicting the main conclusion.
This is incorrect. The answer choices to Weaken questions are rarely assumptions, and if they were, they would be incorrect. Assumptions don't weaken arguments; in fact, the argument depends on them. Furthermore, assumptions are not statements that "could be true but are not even mentioned in the stimulus"; instead, they are implicit premises upon which the conclusion of the argument depends.

In conclusion, you will rarely, if ever, encounter a Weaken question where the correct answer choice directly contradicted a factual premise in the argument. Virtually every single Weaken question asks you to consider the argument critically, and evaluate the implications of the answer choices on the conclusion. The correct answer choice will bring in outside information that, if true, would show that the conclusion is invalid, i.e. that it does not necessarily follow from the premises. The correct answer choice is unlikely to be factually at odds with the premises for that conclusion; instead, it will bring in a piece of information that the author didn't consider when reaching her conclusion, and which - if true - would render the conclusion suspect.

Thanks!
 elbism
  • Posts: 26
  • Joined: Mar 21, 2016
|
#22590
Thank you for that, that's really helpful. Here's an example of a question

Drug company manager: Our newest product is just not selling. One way to save it would be a new marketing campaign. This would not guarantee success, but it is one chance to save the product, so we should try it"

q stem: which one of the following, if true, most seriously weakens the manager's argument?

(C) the drug company should not undertake a new marketing campaign for its newest product if the campaign has no chance to succeed.


C is the incorrect answer.. why? In other words, stimulus: "it might work, so let's try it" answer choice: "it won't work, so don't do it".. and that's somehow incorrect :-?
 elbism
  • Posts: 26
  • Joined: Mar 21, 2016
|
#22591
Nikki Siclunov wrote:Hi elbism,

Let me add to Robert's excellent response above.

I'm afraid that neither the example you gave, nor your reasoning behind it, are logically consistent. If all the argument stated is this:
Jack should go to school today, because then he will have a good day
... then the only logically valid ways to weaken that argument would be to show one of the following:

1. Counterexamples where Jack has gone to school, but he had a bad day;
2. Counterexamples where Jack didn't go to school, but he had a blast: perhaps Jack has even better days when he stays home and plays video games all day long?

Neither of the answer choices you propose have any bearing on the argument at hand. Neither contradicts the stimulus, and both are irrelevant to the conclusion.

Last, I need to comment on something you said in your last paragraph:
I do understand that we are supposed to take the answer choices as true, as stated "if true..." but usually those answer choices are assumptions that could be true but are not even mentioned in the stimulus so don't have any particular baring on directly contradicting the main conclusion.
This is incorrect. The answer choices to Weaken questions are rarely assumptions, and if they were, they would be incorrect. Assumptions don't weaken arguments; in fact, the argument depends on them. Furthermore, assumptions are not statements that "could be true but are not even mentioned in the stimulus"; instead, they are implicit premises upon which the conclusion of the argument depends.

In conclusion, you will rarely, if ever, encounter a Weaken question where the correct answer choice directly contradicted a factual premise in the argument. Virtually every single Weaken question asks you to consider the argument critically, and evaluate the implications of the answer choices on the conclusion. The correct answer choice will bring in outside information that, if true, would show that the conclusion is invalid, i.e. that it does not necessarily follow from the premises. The correct answer choice is unlikely to be factually at odds with the premises for that conclusion; instead, it will bring in a piece of information that the author didn't consider when reaching her conclusion, and which - if true - would render the conclusion suspect.

Thanks!
Okay, so I suppose this answers the question I just posted, that C is wrong insomuch as it has no baring on the stimulus since it directly and factually disagrees with it?

So then, contradicting statements, as you expressed, do not weaken the stimulus and are for the most part incorrect. Correct?
 Robert Carroll
PowerScore Staff
  • PowerScore Staff
  • Posts: 1819
  • Joined: Dec 06, 2013
|
#22592
elbism,

You are misreading what answer choice (C) says. It does not say "the campaign has no chance to succeed," it says "if the campaign has no chance to succeed." A conditional allows you to conclude the necessary condition if you are sure the sufficient condition is true. Here, the chances of success are unknown, so the conditional is not "triggered" - we cannot be sure of the necessary condition until the sufficient condition is met, so the conditional is irrelevant.

Robert Carroll
 elbism
  • Posts: 26
  • Joined: Mar 21, 2016
|
#22593
Robert Carroll wrote:elbism,

You are misreading what answer choice (C) says. It does not say "the campaign has no chance to succeed," it says "if the campaign has no chance to succeed." A conditional allows you to conclude the necessary condition if you are sure the sufficient condition is true. Here, the chances of success are unknown, so the conditional is not "triggered" - we cannot be sure of the necessary condition until the sufficient condition is met, so the conditional is irrelevant.

Robert Carroll
Ohh okay that makes much more sense. I don't think I was familiar with the concept of 'triggers' in conditionality. Must've missed something in my studies. Thank you.
 Robert Carroll
PowerScore Staff
  • PowerScore Staff
  • Posts: 1819
  • Joined: Dec 06, 2013
|
#22594
elbism wrote:
Robert Carroll wrote:elbism,

You are misreading what answer choice (C) says. It does not say "the campaign has no chance to succeed," it says "if the campaign has no chance to succeed." A conditional allows you to conclude the necessary condition if you are sure the sufficient condition is true. Here, the chances of success are unknown, so the conditional is not "triggered" - we cannot be sure of the necessary condition until the sufficient condition is met, so the conditional is irrelevant.

Robert Carroll
Ohh okay that makes much more sense. I don't think I was familiar with the concept of 'triggers' in conditionality. Must've missed something in my studies. Thank you.
elbism,

Think of it like this - a conditional does not say that a particular state of affairs exists. It says that if one state of affairs exists (sufficient condition), then you can be absolutely, 100% sure that another state of affairs exists (necessary condition).

Thus, a statement like:

"If Smith wins the election, then the price of oil will increase by 5%" does not commit me to either of these two statements:

1. "Smith will win the election."

2. "The price of oil will increase by 5%."

It commits me instead to the following: if the first statement becomes true, the second statement must also be true. If it's not, then my conditional was wrong in the first place. That is what I'm committed to.

If I do not have information that allows me to tell whether the first statement is true or false, the second statement is still unknown as well. Thus, in answer choice (C), because we don't know the sufficient condition is true yet, we can't conclude the necessary condition, so the answer on its own does not weaken the stimulus.

Robert Carroll

Get the most out of your LSAT Prep Plus subscription.

Analyze and track your performance with our Testing and Analytics Package.